ExplanationStarting with choice (A), there are 3 slots on the team, with 5 experienced engineers available for the first slot, 4 for the second slot, and 3 for the third slot. Order doesn’t matter, so divide by 3! to get \(\frac{5 \times 4 \times 3}{3 \times 2 \times 1}= 10\).
Choice (A) is correct. For choice (B), you have 3 apprentice engineers available for 2 slots and once again, order doesn’t matter. \(\frac{3 \times 2}{2 \times 1}= 3\).
Choice (B) is correct and you can eliminate choice (C), which contradicts choice (B).
For choice (D) you have \(\frac{5 \times 4}{1 \times 2}= 10\), so
choice (D) is correct. Choice (E) is incorrect because \(\frac{5 \times 4 \times 3 \times 2}{4 \times 3 \times 2 \times 1}= 5\).
The correct answers are choices (A), (B), and (D).
_________________
SandyIf you found this post useful, please let me know by pressing the Kudos ButtonTry our free Online GRE Test